Twin Primes Conjecture and related problemsGoldbach's conjecture and number of ways in which an even number can be expressed as a sum of two primesTwin, cousin, sexy, … primesDisprove the Twin Prime Conjecture for Exotic PrimesSiamese Twin primesSmallest twin-prime-pair exceeding $10^1000$How to prove twin prime conjecture or Goldbach conjecture if we assume prime distribution is completely random?Proof of minor claim related to the Twin Primes ConjectureA twin prime theorem, and a reformulation of the twin prime conjecture

legal rights of neighbouring property owners

How am I ever going to be able to "vet" 120,000+ lines of Composer PHP code not written by me?

Is rotating a pawn so that it faces a different direction and then moves in that direction technically permitted according to the 2018 FIDE Laws?

Infant Mortality: Aluminum Electrolytic Capacitor short circuit failure

How to end the conversation quickly with head hunter?

Does the patient need to be immobile during the tending action from the Healer feat?

Can I use Moonlace to allow a Lightning Bolt to target a creature with protection from red?

Functions that simply call another function, bad design choice?

Who is the narrator of Star Wars?

How can conflict be conducted between nations when warfare is never an option?

Why do we use cross products in physics?

Why does atmospheric pressure act on us?

Sci-fi novel from 1980s(?) about colony on Mars

Site is accessible by domain in all browsers but Chrome

Why is Brownian motion useful in finance?

Sci fi story featuring aliens with segmented bodies, they lose a pair of legs when they breed

What is the common term to express the barrier of a balcony?

Am I obligated to pass on domain knowledge after being let go?

Prime to Prime: Get all first 25 Prime Numbers using up to 4 Primes

Python Code for List of Month Names starting with current month

Why are Amex fees allowed in the European Union?

Why did they design new connectors for USB?

Do any countries have a pensions system funded entirely by past contributions, rather than current taxes?

If you were to fly an ILS in a knife edge would you receve GS and LOC be fliped?



Twin Primes Conjecture and related problems


Goldbach's conjecture and number of ways in which an even number can be expressed as a sum of two primesTwin, cousin, sexy, … primesDisprove the Twin Prime Conjecture for Exotic PrimesSiamese Twin primesSmallest twin-prime-pair exceeding $10^1000$How to prove twin prime conjecture or Goldbach conjecture if we assume prime distribution is completely random?Proof of minor claim related to the Twin Primes ConjectureA twin prime theorem, and a reformulation of the twin prime conjecture






.everyoneloves__top-leaderboard:empty,.everyoneloves__mid-leaderboard:empty,.everyoneloves__bot-mid-leaderboard:empty
margin-bottom:0;

.everyonelovesstackoverflowposition:absolute;height:1px;width:1px;opacity:0;top:0;left:0;pointer-events:none;








4















$begingroup$


I am wondering if my approach below, based on probability theory and thus heuristic (it is by no means a proof) is new and worth pursuing. Applications are numerous.



Let $S_f$ be a random set of positive integers defined as follows: the positive integer $n$ belongs to $S_f$ with probability $f(n)$. In other words, for each integer $n$, generate a uniform deviate $U_n$ on $[0, 1]$. If and only if $U_n < f(n)$ then add $n$ to $S_f$. The deviates $U_n$ are independent. Now consider $f(n) = frac1log n$ if $n > 2$, and $f(1)= f(2) = 0$. With this particular choice of $f$, the set $S_f$ has the same density as the set of prime numbers, and behaves in a very similar way. Let us define the following functions (actually, they are random variables):



  • $M(n)$ is the number of elements in $S_f$ that are smaller than $n$.


  • $T(n)$ is the number of twin pairs $(k, k+1)$ with both $k, k+1 in
    S_f$
    and $k leq n$.


The twins in $S_f$ play the same role as twin primes. It is is easy to prove that $M(n)$ (its expectation) is asymptotically equal to the logarithm integral $mboxLi(n)$. The same is true for the prime number counting function. Furthermore, the prime numbers themselves is a particular case of $S_f$, a particular realization of the random set $S_f$ for the same function $f$. (you need to ignore even integers in the construction process, but other than that, it's straightforward.)



Then we also have:
$$mboxE[T(n)] sim Csum_k=1^n-1 f(k)f(k+1)sim C'int_2^n fracdt(log t)^2 .$$



Here E denotes the expectation. The same result is conjectured to be true for twin primes. It is easy to show that there are infinitely many twin numbers in $S_f$, and assess whether the series consisting of the reciprocals of these twins, converge. The same arguments could be applied to twin primes, but of course, it would no constitute a proof, just some heuristic argumentation. Are there any useful references related to the approach discussed here?



Update: I plan on using the same methodology for the Goldbach conjecture, and in particular to obtain an asymptotic value for the expected number of ways an even number $n$ can be represented as a sum of two primes. With my notation, it should be asymptotically equivalent to
$$sum_k=1^lfloor n/2 rfloor f(k)f(n-k) sim fracn2(log n)^2.$$
See also here and here.










share|cite|improve this question











$endgroup$














  • $begingroup$
    I wrote an article about twins prime conjecture (k-tuple conjecture in general) without using probability theory : mathoverflow.net/questions/338247/…
    $endgroup$
    – LAGRIDA
    Sep 29 at 13:09

















4















$begingroup$


I am wondering if my approach below, based on probability theory and thus heuristic (it is by no means a proof) is new and worth pursuing. Applications are numerous.



Let $S_f$ be a random set of positive integers defined as follows: the positive integer $n$ belongs to $S_f$ with probability $f(n)$. In other words, for each integer $n$, generate a uniform deviate $U_n$ on $[0, 1]$. If and only if $U_n < f(n)$ then add $n$ to $S_f$. The deviates $U_n$ are independent. Now consider $f(n) = frac1log n$ if $n > 2$, and $f(1)= f(2) = 0$. With this particular choice of $f$, the set $S_f$ has the same density as the set of prime numbers, and behaves in a very similar way. Let us define the following functions (actually, they are random variables):



  • $M(n)$ is the number of elements in $S_f$ that are smaller than $n$.


  • $T(n)$ is the number of twin pairs $(k, k+1)$ with both $k, k+1 in
    S_f$
    and $k leq n$.


The twins in $S_f$ play the same role as twin primes. It is is easy to prove that $M(n)$ (its expectation) is asymptotically equal to the logarithm integral $mboxLi(n)$. The same is true for the prime number counting function. Furthermore, the prime numbers themselves is a particular case of $S_f$, a particular realization of the random set $S_f$ for the same function $f$. (you need to ignore even integers in the construction process, but other than that, it's straightforward.)



Then we also have:
$$mboxE[T(n)] sim Csum_k=1^n-1 f(k)f(k+1)sim C'int_2^n fracdt(log t)^2 .$$



Here E denotes the expectation. The same result is conjectured to be true for twin primes. It is easy to show that there are infinitely many twin numbers in $S_f$, and assess whether the series consisting of the reciprocals of these twins, converge. The same arguments could be applied to twin primes, but of course, it would no constitute a proof, just some heuristic argumentation. Are there any useful references related to the approach discussed here?



Update: I plan on using the same methodology for the Goldbach conjecture, and in particular to obtain an asymptotic value for the expected number of ways an even number $n$ can be represented as a sum of two primes. With my notation, it should be asymptotically equivalent to
$$sum_k=1^lfloor n/2 rfloor f(k)f(n-k) sim fracn2(log n)^2.$$
See also here and here.










share|cite|improve this question











$endgroup$














  • $begingroup$
    I wrote an article about twins prime conjecture (k-tuple conjecture in general) without using probability theory : mathoverflow.net/questions/338247/…
    $endgroup$
    – LAGRIDA
    Sep 29 at 13:09













4













4









4





$begingroup$


I am wondering if my approach below, based on probability theory and thus heuristic (it is by no means a proof) is new and worth pursuing. Applications are numerous.



Let $S_f$ be a random set of positive integers defined as follows: the positive integer $n$ belongs to $S_f$ with probability $f(n)$. In other words, for each integer $n$, generate a uniform deviate $U_n$ on $[0, 1]$. If and only if $U_n < f(n)$ then add $n$ to $S_f$. The deviates $U_n$ are independent. Now consider $f(n) = frac1log n$ if $n > 2$, and $f(1)= f(2) = 0$. With this particular choice of $f$, the set $S_f$ has the same density as the set of prime numbers, and behaves in a very similar way. Let us define the following functions (actually, they are random variables):



  • $M(n)$ is the number of elements in $S_f$ that are smaller than $n$.


  • $T(n)$ is the number of twin pairs $(k, k+1)$ with both $k, k+1 in
    S_f$
    and $k leq n$.


The twins in $S_f$ play the same role as twin primes. It is is easy to prove that $M(n)$ (its expectation) is asymptotically equal to the logarithm integral $mboxLi(n)$. The same is true for the prime number counting function. Furthermore, the prime numbers themselves is a particular case of $S_f$, a particular realization of the random set $S_f$ for the same function $f$. (you need to ignore even integers in the construction process, but other than that, it's straightforward.)



Then we also have:
$$mboxE[T(n)] sim Csum_k=1^n-1 f(k)f(k+1)sim C'int_2^n fracdt(log t)^2 .$$



Here E denotes the expectation. The same result is conjectured to be true for twin primes. It is easy to show that there are infinitely many twin numbers in $S_f$, and assess whether the series consisting of the reciprocals of these twins, converge. The same arguments could be applied to twin primes, but of course, it would no constitute a proof, just some heuristic argumentation. Are there any useful references related to the approach discussed here?



Update: I plan on using the same methodology for the Goldbach conjecture, and in particular to obtain an asymptotic value for the expected number of ways an even number $n$ can be represented as a sum of two primes. With my notation, it should be asymptotically equivalent to
$$sum_k=1^lfloor n/2 rfloor f(k)f(n-k) sim fracn2(log n)^2.$$
See also here and here.










share|cite|improve this question











$endgroup$




I am wondering if my approach below, based on probability theory and thus heuristic (it is by no means a proof) is new and worth pursuing. Applications are numerous.



Let $S_f$ be a random set of positive integers defined as follows: the positive integer $n$ belongs to $S_f$ with probability $f(n)$. In other words, for each integer $n$, generate a uniform deviate $U_n$ on $[0, 1]$. If and only if $U_n < f(n)$ then add $n$ to $S_f$. The deviates $U_n$ are independent. Now consider $f(n) = frac1log n$ if $n > 2$, and $f(1)= f(2) = 0$. With this particular choice of $f$, the set $S_f$ has the same density as the set of prime numbers, and behaves in a very similar way. Let us define the following functions (actually, they are random variables):



  • $M(n)$ is the number of elements in $S_f$ that are smaller than $n$.


  • $T(n)$ is the number of twin pairs $(k, k+1)$ with both $k, k+1 in
    S_f$
    and $k leq n$.


The twins in $S_f$ play the same role as twin primes. It is is easy to prove that $M(n)$ (its expectation) is asymptotically equal to the logarithm integral $mboxLi(n)$. The same is true for the prime number counting function. Furthermore, the prime numbers themselves is a particular case of $S_f$, a particular realization of the random set $S_f$ for the same function $f$. (you need to ignore even integers in the construction process, but other than that, it's straightforward.)



Then we also have:
$$mboxE[T(n)] sim Csum_k=1^n-1 f(k)f(k+1)sim C'int_2^n fracdt(log t)^2 .$$



Here E denotes the expectation. The same result is conjectured to be true for twin primes. It is easy to show that there are infinitely many twin numbers in $S_f$, and assess whether the series consisting of the reciprocals of these twins, converge. The same arguments could be applied to twin primes, but of course, it would no constitute a proof, just some heuristic argumentation. Are there any useful references related to the approach discussed here?



Update: I plan on using the same methodology for the Goldbach conjecture, and in particular to obtain an asymptotic value for the expected number of ways an even number $n$ can be represented as a sum of two primes. With my notation, it should be asymptotically equivalent to
$$sum_k=1^lfloor n/2 rfloor f(k)f(n-k) sim fracn2(log n)^2.$$
See also here and here.







sequences-and-series number-theory probability-theory prime-numbers






share|cite|improve this question















share|cite|improve this question













share|cite|improve this question




share|cite|improve this question








edited Sep 29 at 12:42







Vincent Granville

















asked Sep 29 at 0:42









Vincent GranvilleVincent Granville

2,0054 silver badges18 bronze badges




2,0054 silver badges18 bronze badges














  • $begingroup$
    I wrote an article about twins prime conjecture (k-tuple conjecture in general) without using probability theory : mathoverflow.net/questions/338247/…
    $endgroup$
    – LAGRIDA
    Sep 29 at 13:09
















  • $begingroup$
    I wrote an article about twins prime conjecture (k-tuple conjecture in general) without using probability theory : mathoverflow.net/questions/338247/…
    $endgroup$
    – LAGRIDA
    Sep 29 at 13:09















$begingroup$
I wrote an article about twins prime conjecture (k-tuple conjecture in general) without using probability theory : mathoverflow.net/questions/338247/…
$endgroup$
– LAGRIDA
Sep 29 at 13:09




$begingroup$
I wrote an article about twins prime conjecture (k-tuple conjecture in general) without using probability theory : mathoverflow.net/questions/338247/…
$endgroup$
– LAGRIDA
Sep 29 at 13:09










1 Answer
1






active

oldest

votes


















7

















$begingroup$

You're about 90 years too late. The probabilistic approach to prime numbers was developed by Cramér in the 1930's. You might look at this paper by the other Granville.






share|cite|improve this answer










$endgroup$









  • 2




    $begingroup$
    I am writing a book on probabilistic number theory, I don't want to mention something is new if it is not, thus the reason for my question. The Wikipedia entry (see en.wikipedia.org/wiki/Twin_prime) on Twin Primes mentions that " The conjecture can be justified (but not proven) by assuming that 1 / ln t describes the density function of the prime distribution, an assumption suggested by the prime number theorem and would imply the twin prime conjecture, but remains unresolved." but with no references and no explanation.
    $endgroup$
    – Vincent Granville
    Sep 29 at 2:01






  • 2




    $begingroup$
    Here is a PDF version of that document: dartmouth.edu/~chance/chance_news/for_chance_news/Riemann/…
    $endgroup$
    – Vincent Granville
    Sep 29 at 2:10












Your Answer








StackExchange.ready(function()
var channelOptions =
tags: "".split(" "),
id: "69"
;
initTagRenderer("".split(" "), "".split(" "), channelOptions);

StackExchange.using("externalEditor", function()
// Have to fire editor after snippets, if snippets enabled
if (StackExchange.settings.snippets.snippetsEnabled)
StackExchange.using("snippets", function()
createEditor();
);

else
createEditor();

);

function createEditor()
StackExchange.prepareEditor(
heartbeatType: 'answer',
autoActivateHeartbeat: false,
convertImagesToLinks: true,
noModals: true,
showLowRepImageUploadWarning: true,
reputationToPostImages: 10,
bindNavPrevention: true,
postfix: "",
imageUploader:
brandingHtml: "Powered by u003ca class="icon-imgur-white" href="https://imgur.com/"u003eu003c/au003e",
contentPolicyHtml: "User contributions licensed under u003ca href="https://creativecommons.org/licenses/by-sa/4.0/"u003ecc by-sa 4.0 with attribution requiredu003c/au003e u003ca href="https://stackoverflow.com/legal/content-policy"u003e(content policy)u003c/au003e",
allowUrls: true
,
noCode: true, onDemand: true,
discardSelector: ".discard-answer"
,immediatelyShowMarkdownHelp:true
);



);














draft saved

draft discarded
















StackExchange.ready(
function ()
StackExchange.openid.initPostLogin('.new-post-login', 'https%3a%2f%2fmath.stackexchange.com%2fquestions%2f3373731%2ftwin-primes-conjecture-and-related-problems%23new-answer', 'question_page');

);

Post as a guest















Required, but never shown


























1 Answer
1






active

oldest

votes








1 Answer
1






active

oldest

votes









active

oldest

votes






active

oldest

votes









7

















$begingroup$

You're about 90 years too late. The probabilistic approach to prime numbers was developed by Cramér in the 1930's. You might look at this paper by the other Granville.






share|cite|improve this answer










$endgroup$









  • 2




    $begingroup$
    I am writing a book on probabilistic number theory, I don't want to mention something is new if it is not, thus the reason for my question. The Wikipedia entry (see en.wikipedia.org/wiki/Twin_prime) on Twin Primes mentions that " The conjecture can be justified (but not proven) by assuming that 1 / ln t describes the density function of the prime distribution, an assumption suggested by the prime number theorem and would imply the twin prime conjecture, but remains unresolved." but with no references and no explanation.
    $endgroup$
    – Vincent Granville
    Sep 29 at 2:01






  • 2




    $begingroup$
    Here is a PDF version of that document: dartmouth.edu/~chance/chance_news/for_chance_news/Riemann/…
    $endgroup$
    – Vincent Granville
    Sep 29 at 2:10















7

















$begingroup$

You're about 90 years too late. The probabilistic approach to prime numbers was developed by Cramér in the 1930's. You might look at this paper by the other Granville.






share|cite|improve this answer










$endgroup$









  • 2




    $begingroup$
    I am writing a book on probabilistic number theory, I don't want to mention something is new if it is not, thus the reason for my question. The Wikipedia entry (see en.wikipedia.org/wiki/Twin_prime) on Twin Primes mentions that " The conjecture can be justified (but not proven) by assuming that 1 / ln t describes the density function of the prime distribution, an assumption suggested by the prime number theorem and would imply the twin prime conjecture, but remains unresolved." but with no references and no explanation.
    $endgroup$
    – Vincent Granville
    Sep 29 at 2:01






  • 2




    $begingroup$
    Here is a PDF version of that document: dartmouth.edu/~chance/chance_news/for_chance_news/Riemann/…
    $endgroup$
    – Vincent Granville
    Sep 29 at 2:10













7















7











7







$begingroup$

You're about 90 years too late. The probabilistic approach to prime numbers was developed by Cramér in the 1930's. You might look at this paper by the other Granville.






share|cite|improve this answer










$endgroup$



You're about 90 years too late. The probabilistic approach to prime numbers was developed by Cramér in the 1930's. You might look at this paper by the other Granville.







share|cite|improve this answer













share|cite|improve this answer




share|cite|improve this answer










answered Sep 29 at 1:43









Robert IsraelRobert Israel

355k23 gold badges246 silver badges514 bronze badges




355k23 gold badges246 silver badges514 bronze badges










  • 2




    $begingroup$
    I am writing a book on probabilistic number theory, I don't want to mention something is new if it is not, thus the reason for my question. The Wikipedia entry (see en.wikipedia.org/wiki/Twin_prime) on Twin Primes mentions that " The conjecture can be justified (but not proven) by assuming that 1 / ln t describes the density function of the prime distribution, an assumption suggested by the prime number theorem and would imply the twin prime conjecture, but remains unresolved." but with no references and no explanation.
    $endgroup$
    – Vincent Granville
    Sep 29 at 2:01






  • 2




    $begingroup$
    Here is a PDF version of that document: dartmouth.edu/~chance/chance_news/for_chance_news/Riemann/…
    $endgroup$
    – Vincent Granville
    Sep 29 at 2:10












  • 2




    $begingroup$
    I am writing a book on probabilistic number theory, I don't want to mention something is new if it is not, thus the reason for my question. The Wikipedia entry (see en.wikipedia.org/wiki/Twin_prime) on Twin Primes mentions that " The conjecture can be justified (but not proven) by assuming that 1 / ln t describes the density function of the prime distribution, an assumption suggested by the prime number theorem and would imply the twin prime conjecture, but remains unresolved." but with no references and no explanation.
    $endgroup$
    – Vincent Granville
    Sep 29 at 2:01






  • 2




    $begingroup$
    Here is a PDF version of that document: dartmouth.edu/~chance/chance_news/for_chance_news/Riemann/…
    $endgroup$
    – Vincent Granville
    Sep 29 at 2:10







2




2




$begingroup$
I am writing a book on probabilistic number theory, I don't want to mention something is new if it is not, thus the reason for my question. The Wikipedia entry (see en.wikipedia.org/wiki/Twin_prime) on Twin Primes mentions that " The conjecture can be justified (but not proven) by assuming that 1 / ln t describes the density function of the prime distribution, an assumption suggested by the prime number theorem and would imply the twin prime conjecture, but remains unresolved." but with no references and no explanation.
$endgroup$
– Vincent Granville
Sep 29 at 2:01




$begingroup$
I am writing a book on probabilistic number theory, I don't want to mention something is new if it is not, thus the reason for my question. The Wikipedia entry (see en.wikipedia.org/wiki/Twin_prime) on Twin Primes mentions that " The conjecture can be justified (but not proven) by assuming that 1 / ln t describes the density function of the prime distribution, an assumption suggested by the prime number theorem and would imply the twin prime conjecture, but remains unresolved." but with no references and no explanation.
$endgroup$
– Vincent Granville
Sep 29 at 2:01




2




2




$begingroup$
Here is a PDF version of that document: dartmouth.edu/~chance/chance_news/for_chance_news/Riemann/…
$endgroup$
– Vincent Granville
Sep 29 at 2:10




$begingroup$
Here is a PDF version of that document: dartmouth.edu/~chance/chance_news/for_chance_news/Riemann/…
$endgroup$
– Vincent Granville
Sep 29 at 2:10


















draft saved

draft discarded















































Thanks for contributing an answer to Mathematics Stack Exchange!


  • Please be sure to answer the question. Provide details and share your research!

But avoid


  • Asking for help, clarification, or responding to other answers.

  • Making statements based on opinion; back them up with references or personal experience.

Use MathJax to format equations. MathJax reference.


To learn more, see our tips on writing great answers.




draft saved


draft discarded














StackExchange.ready(
function ()
StackExchange.openid.initPostLogin('.new-post-login', 'https%3a%2f%2fmath.stackexchange.com%2fquestions%2f3373731%2ftwin-primes-conjecture-and-related-problems%23new-answer', 'question_page');

);

Post as a guest















Required, but never shown





















































Required, but never shown














Required, but never shown












Required, but never shown







Required, but never shown

































Required, but never shown














Required, but never shown












Required, but never shown







Required, but never shown









Popular posts from this blog

Tamil (spriik) Luke uk diar | Nawigatjuun

Align equal signs while including text over equalitiesAMS align: left aligned text/math plus multicolumn alignmentMultiple alignmentsAligning equations in multiple placesNumbering and aligning an equation with multiple columnsHow to align one equation with another multline equationUsing \ in environments inside the begintabularxNumber equations and preserving alignment of equal signsHow can I align equations to the left and to the right?Double equation alignment problem within align enviromentAligned within align: Why are they right-aligned?

Where does the image of a data connector as a sharp metal spike originate from?Where does the concept of infected people turning into zombies only after death originate from?Where does the motif of a reanimated human head originate?Where did the notion that Dragons could speak originate?Where does the archetypal image of the 'Grey' alien come from?Where did the suffix '-Man' originate?Where does the notion of being injured or killed by an illusion originate?Where did the term “sophont” originate?Where does the trope of magic spells being driven by advanced technology originate from?Where did the term “the living impaired” originate?